Difference between revisions of "2013 AMC 8 Problems/Problem 1"

(Solution)
(Solution)
(13 intermediate revisions by 7 users not shown)
Line 1: Line 1:
 
==Problem==
 
==Problem==
 +
Danica wants to arrange her model cars in rows with exactly 6 cars in each row. She now has 23 model cars. What is the smallest number of additional cars she must buy in order to be able to arrange all her cars this way?
 +
 +
<math>\textbf{(A)}\ 1 \qquad \textbf{(B)}\ 2 \qquad \textbf{(C)}\ 3 \qquad \textbf{(D)}\ 4 \qquad \textbf{(E)}\ 5</math>
  
 
==Solution==
 
==Solution==
<math><math>Insert formula here</math><math><math>Insert formula here</math><math><math>Insert formula here</math><math><math>Insert formula here</math><math><math>Insert formula here</math></math></math></math></math></math>
+
The least multiple of 6 greater than 23 is 24. So she will need to add <math>\boxed{\textbf{(A)}\ 1}</math> more model car. ~avamarora
  
 
==See Also==
 
==See Also==
 
{{AMC8 box|year=2013|before=First Problem|num-a=2}}
 
{{AMC8 box|year=2013|before=First Problem|num-a=2}}
 
{{MAA Notice}}
 
{{MAA Notice}}

Revision as of 13:45, 1 November 2020

Problem

Danica wants to arrange her model cars in rows with exactly 6 cars in each row. She now has 23 model cars. What is the smallest number of additional cars she must buy in order to be able to arrange all her cars this way?

$\textbf{(A)}\ 1 \qquad \textbf{(B)}\ 2 \qquad \textbf{(C)}\ 3 \qquad \textbf{(D)}\ 4 \qquad \textbf{(E)}\ 5$

Solution

The least multiple of 6 greater than 23 is 24. So she will need to add $\boxed{\textbf{(A)}\ 1}$ more model car. ~avamarora

See Also

2013 AMC 8 (ProblemsAnswer KeyResources)
Preceded by
First Problem
Followed by
Problem 2
1 2 3 4 5 6 7 8 9 10 11 12 13 14 15 16 17 18 19 20 21 22 23 24 25
All AJHSME/AMC 8 Problems and Solutions

The problems on this page are copyrighted by the Mathematical Association of America's American Mathematics Competitions. AMC logo.png